10
$\begingroup$

I am trying to write a Monte Carlo simulation to calculate risk associated with some property based products. What is the most reasonable stochastic process to model property price index? Do people simulate is together with some other factors? (GDP, consumer confidence, stock prices, etc.)?

Because I am after risk, I need to work in the real measure (as opposed to risk-neutral).

Also this is needed for academic work, so please mention any citable materials if relevant.

$\endgroup$
1
  • 1
    $\begingroup$ Hi Grzenio, welcome to quant.SE and thanks for posting your question. Models of property prices have not been discussed here before to my knowledge, so hopefully there are some knowledgeable folks around. Thanks for broadening the range of questions. $\endgroup$ Dec 20, 2011 at 22:47

1 Answer 1

6
$\begingroup$

The best I have seen so far is William Wheaton's work in this area. I don't know how much is described in his papers but he and Torto created a system that combined factor models for things like local and national price indexes with specific economics of commercial real estate ventures (such as balloon payments on construction milestones and the like).

The whole thing was probably overparameterized, but pretty neat.

Obviously recreating that whole thing would be a huge burden for you, but the idea of tying a factor model to your favorite GDP predictors and so forth is fairly reasonable, especially for academic research. (Actual investors are more likely to code a few hunches into some scenarios in a spreadsheet and leave it at that).

$\endgroup$
2
  • $\begingroup$ Could you point me to the actual paper? I found this: econ-www.mit.edu/files/2390, but it is about the return on properties rather than their actual prices. $\endgroup$
    – Grzenio
    Dec 30, 2011 at 13:32
  • $\begingroup$ The documentation we saw for the model was privately shared, because his consultancy was trying to sell us a whole system. I've changed firms since then and do not have a pointer, I am afraid. $\endgroup$
    – Brian B
    Jan 3, 2012 at 13:27

Your Answer

By clicking “Post Your Answer”, you agree to our terms of service and acknowledge you have read our privacy policy.

Not the answer you're looking for? Browse other questions tagged or ask your own question.